Math Is Fun Forum

  Discussion about math, puzzles, games and fun.   Useful symbols: ÷ × ½ √ ∞ ≠ ≤ ≥ ≈ ⇒ ± ∈ Δ θ ∴ ∑ ∫ • π ƒ -¹ ² ³ °

You are not logged in.

#1 2009-02-08 21:34:32

chriss
Member
Registered: 2009-02-08
Posts: 4

limits

lim (x[sup]χ[/sup][sup]1/2[/sup] ) /  3[sup]χ[/sup],
x->+infinity
can you help me to solve this?

Offline

#2 2009-02-15 02:36:25

coffeeking
Member
Registered: 2007-11-18
Posts: 44

Re: limits

Bump this up since I am interested in knowing too...

Offline

#3 2009-02-15 03:42:33

Ricky
Moderator
Registered: 2005-12-04
Posts: 3,791

Re: limits

Have you tried L'Hospital's rule?


"In the real world, this would be a problem.  But in mathematics, we can just define a place where this problem doesn't exist.  So we'll go ahead and do that now..."

Offline

#4 2009-02-19 07:11:16

janetpc2008
Member
Registered: 2009-02-03
Posts: 7

Re: limits

Does it help if you just look at squares of 3? This way the fraction can be combined.
Define a subset of x:


So now we have 3 to a power in the numerator and the denominator, so we can subtract them to get the fraction:

In the limit of y going to +infinity, is this easier to manage?

I'm not sure how to prove this goes to zero, but it does.

Offline

#5 2009-02-19 08:32:31

JaneFairfax
Member
Registered: 2007-02-23
Posts: 6,868

Re: limits

If you can prove that

as
, then
.


Ricky wrote:

Have you tried L'Hôpital's rule?

I did, and got stuck. BangHead.gif

Last edited by JaneFairfax (2009-02-23 03:58:01)

Offline

#6 2009-02-22 21:12:36

Eigma
Member
Registered: 2009-02-22
Posts: 5

Re: limits

Hi. I'm really confused how to start proving sequences. Can you give me a technique, for example, how to prove that if an approaches A and bn approaches B, then an/bn approaches A/B where bn and B is not 0.


"A smile is a curve that can set things straight."

Offline

#7 2009-02-23 01:16:49

LampShade
Member
Registered: 2009-02-22
Posts: 23

Re: limits

Thanks for the tip about /displaystyle

Last edited by LampShade (2009-02-23 06:06:38)


--  Boozer

Offline

#8 2009-02-23 03:56:59

JaneFairfax
Member
Registered: 2007-02-23
Posts: 6,868

Re: limits

This happens when you use $$. Use \displaystyle to restore settings.

\mbox{Let } y = \frac {x^{\sqrt{x}}}{3^x} \\
\ln{y} =\sqrt{x} \ln{x} - x \ln{3}  \\
\ln{y} = x ( $$\frac {\ln{x}}{\sqrt{x}}$$ - \ln{3} ) \\
\\
\mbox{Use L'Hospital's rule to show that} \displaystyle\lim_{x \to \infty} \mbox {of the first term is zero, and so the overall limit goes to } -\infty \mbox{.  Hence the limit you seek is 0.}

Offline

Board footer

Powered by FluxBB